Questions tagged «bounds»

3
仅了解最小值/最大值的数据的统计方法
是否有统计信息的一个分支来处理其确切值未知的数据,但是对于每个人,我们都知道该值的最大值或最小值? I suspect that my problem stems largely from the fact that I am struggling to articulate it in statistical terms, but hopefully an example will help to clarify: 假设存在两个相互连接的总体AAA和BBB,以便在某个时候成员AAA可以“转换”为BBB,但不可能相反。过渡时间是可变的,但不是随机的。例如,AAA可以是“没有后代的个体”,而BBB “至少有一个后代的个体”。我对这种进展发生的年龄感兴趣,但我只有横截面数据。对于任何给定的个体,我可以找出它们是否属于AAA或BBB。我也知道这些人的年龄。对于人群A中的每个人AAA,我知道过渡年龄将比其当前年龄更大。同样,对于B的成员BBB,我知道过渡年龄比当前年龄小。但是我不知道确切的值。 假设我还有其他一些要与过渡年龄进行比较的因素。例如,我想知道一个人的亚种或体型是否会影响第一个后代的年龄。我绝对有一些有用的信息可以回答这些问题:平均而言,在中的个体中AAA,年龄较大的个体将有一个较晚的过渡期。但是这些信息并不完美,特别是对于年轻人而言。反之亦然人口BBB。 Are there established methods to deal with this sort of data? I do not necessarily need a full …

2
我们如何限制随机变量最大的概率?
\newcommand{\P}{\mathbb{P}}假设我们有NNN独立的随机变量X1X1X_1,……\ldots,XnXnX_n具有有限的均值μ1≤…≤μNμ1≤…≤μN\mu_1 \leq \ldots \leq \mu_N和方差σ21σ12\sigma_1^2,……\ldots,σ2NσN2\sigma_N^2。我正在寻找任意Xi≠XNXi≠XNX_i \neq X_N大于所有其他XjXjX_j,j \ neq i的概率的无分布边界j≠ij≠ij \neq i。 换句话说,如果为了简单起见,我们假设X_i的分布XiXiX_i是连续的(使得P(Xi=Xj)=0P(Xi=Xj)=0\P(X_i = X_j) = 0),那么我正在寻找 P(Xi=maxjXj).P(Xi=maxjXj). \P( X_i = \max_j X_j ) \enspace. 如果N=2N=2N=2,我们可以使用切比雪夫不等式得到: P(X1=maxjXj)=P(X1>X2)≤σ21+σ22σ21+σ22+(μ1−μ2)2.P(X1=maxjXj)=P(X1>X2)≤σ12+σ22σ12+σ22+(μ1−μ2)2. \P(X_1 = \max_j X_j) = \P(X_1 > X_2) \leq \frac{\sigma_1^2 + \sigma_2^2}{\sigma_1^2 + \sigma_2^2 + (\mu_1 - \mu_2)^2} \enspace. 我想找到一般N的一些简单(不一定紧)边界NNN,但是我无法找到(美学上)一般N的令人满意的结果NNN。 请注意,这些变量不假定为iid。欢迎对相关工作提出任何建议或参考。 更新:回想一下,假设μj≥μiμj≥μi\mu_j \geq …



1
copula密度的上限?
的Fréchet可-Hoeffding上限适用于连接函数分布函数,它是由下式给出 C(u1,...,ud)≤min{u1,..,ud}.C(u1,...,ud)≤min{u1,..,ud}.C(u_1,...,u_d)\leq \min\{u_1,..,u_d\}. 系密度而不是CDF 是否有相似的上限(在某种程度上取决于边际密度)?c(u1,...,ud)c(u1,...,ud)c(u_1,...,u_d) 任何参考将不胜感激。


2
样本最大值的方差是多少?
我正在寻找一组随机变量的最大值的方差的界限。换句话说,我想找的闭合形式的公式乙BB,使得 其中 X = { X 1,... ,X 中号 }是一组固定的中号与有限装置的随机变量 μ 1,... ,μ 中号和方差 σ 2 1,... ,σ 2 中号。瓦尔(最大一世X一世)≤ 乙,Var(maxiXi)≤B, \mbox{Var}(\max_i X_i) \leq B \enspace, X= { X1个,… ,X中号}X={X1,…,XM}X = \{ X_1, \ldots, X_M \}中号MMμ1个,… ,μ中号μ1,…,μM\mu_1, \ldots, \mu_Mσ21个,… ,σ2中号σ12,…,σM2\sigma_1^2, \ldots, \sigma_M^2 我可以推断出 但是这个界限似乎很松散。数值试验似乎表明,乙= 最大值我σ 2 我可能是一种可能性,但我一直没能证明这一点。任何帮助表示赞赏。瓦尔(最大一世X一世)≤ Σ一世σ2一世,Var(maxiXi)≤∑iσi2, \mbox{Var}(\max_i X_i) …

1
预期平均值将超过一个值的预期次数
给定一系列iid随机变量,例如,对于,为,我试图限制经验均值的预期次数会超过一个值,因为我们继续绘制样本,即: Xi∈[0,1]Xi∈[0,1]X_i \in [0,1]i=1,2,...,ni=1,2,...,ni = 1,2,...,n1n∑ni=1Xi1n∑i=1nXi\frac{1}{n}\sum_{i=1}^n X_ic≥0c≥0c \geq 0T=def∑j=1nP({1j∑i=1jXi≥c})T=def∑j=1nP({1j∑i=1jXi≥c}) \mathcal{T} \overset{def}{=} \sum_{j=1}^n \mathbb{P} \left(\left\{ \frac{1}{j}\sum_{i=1}^j X_i \geq c\right\}\right) 如果我们假设对于,我们可以使用Hoeffding不等式得出c=a+E[X]c=a+E[X]c = a + \mathbb{E}[X]a>0a>0a > 0 T≤∑j=1ne−2ja2=1−e−2a2ne2a2−1T≤∑j=1ne−2ja2=1−e−2a2ne2a2−1\begin{align} \mathcal{T} & \leq \sum_{j=1}^n e^{-2ja^2} \\ & = \frac{1 - e^{-2 a^2 n}}{e^{2 a^2}-1} \end{align} 哪个看起来不错(也许),但实际上是一个松散的界限,是否有更好的方法来限制此值?我希望可能会有一种方法,因为不同的事件(每个)显然不是独立的,我不知道有任何方法可以利用这种依赖性。同样,最好删除大于平均值的限制。jjjccc 编辑:如果我们使用马尔可夫不等式,可以消除对大于均值的限制:ccc T≤∑j=1n1jE[X]c=E[X]HncT≤∑j=1n1jE[X]c=E[X]Hnc\begin{align} \mathcal{T} & \leq \sum_{j=1}^n \frac{\frac{1}{j}\mathbb{E}[X]}{c} \\ …

2
欧几里得模上的尾边界,用于在
关于统一选择元素的欧几里得范数多久的已知上限 {−n, −(n−1), ..., n−1, n}d{−n, −(n−1), ..., n−1, n}d\:\{-n,~-(n-1),~...,~n-1,~n\}^d\: 将大于给定的阈值? 我主要对当nnn远小于时以指数收敛到零的范围感兴趣ddd。

1
处理异常有限的响应变量的回归
我正在尝试对理论上限制在-225和+225之间的响应变量进行建模。变量是受试者在玩游戏时获得的总分。尽管从理论上讲,受试者有可能得分+225。尽管如此,这是因为得分不仅取决于对象的动作,而且还取决于其他动作的动作,任何人得分最多为125(这是两个互相玩的最高玩家都可以得分),这种情况的发生频率很高。最低分数是+35。 125的边界导致线性回归困难。我唯一想做的就是将响应重新缩放为0到1之间并使用beta回归。如果我这样做,虽然不确定,我真的可以说125是最高边界(或转换后的1),因为它有可能得分+225。此外,如果我这样做了,我的下界35是什么? 谢谢, 乔纳森

2
假设检验和总变异距离与Kullback-Leibler散度的关系
在我的研究中,我遇到了以下一般性问题:在同一个域中有两个分布和,以及来自这些分布的大量(但有限)样本。样本是从这两个分布之一独立且相同地分布的(尽管分布可能是相关的:例如,可能是和其他分布的混合。)零假设是样本来自,替代假设是样本来自。Q Q P P QPPPQQQQQQPPPPPPQQQ 我试图表征I型和测试样品,了解发行第二类错误和。特别是,除了对和的了解之外,我还对限制一个错误和另一个错误感兴趣。Q P QPPPQQQPPPQQQ 我问了一个关于math.SE 的问题,关于和之间的总变异距离与假设检验的关系,并收到了我接受的答案。这个答案是有道理的,但是我仍然无法将总变化距离和假设检验之间更深层的含义笼罩在脑海中,因为这与我的问题有关。因此,我决定转向这个论坛。QPPPQQQ 我的第一个问题是:总变化是否与 I类错误和II类错误的概率之和无关,而与所采用的假设检验方法无关?本质上,只要存在可能由任一分布生成样本的非零概率,至少一个错误的概率就必须为非零。基本上,无论您进行多少信号处理,您都无法避免假设检验器会出错的可能性。而总变化限制了确切的可能性。我的理解正确吗? I型和II型错误与潜在的概率分布和之间还有另一关系:KL散度。因此,我的第二个问题是:KL散度约束是否仅适用于一种特定的假设检验方法(似乎很多涉及对数似然比方法),还是可以将其普遍适用于所有假设检验方法?如果它适用于所有假设检验方法,那么为什么它似乎与总变异范围有很大不同?它的行为是否有所不同?QPPPQQQ 我的基本问题是:在规定的条件下我应该使用约束还是纯粹为了方便起见?什么时候应该使用一个绑定推导结果并使用另一个绑定? 如果这些问题无关紧要,我深表歉意。我是计算机科学家(所以对我来说,这似乎是一个奇特的模式匹配问题:)。)我对信息论非常了解,并且也具有概率论的毕业背景。但是,我才刚刚开始学习所有这些假设检验的知识。如果需要,我将尽力澄清我的问题。

1
相关随机变量之差的界线
给定两个高度相关的随机变量和,我想限制差的概率超出一定数量: XXXYYY|X−Y||X−Y| |X - Y| P(|X−Y|&gt;K)&lt;δP(|X−Y|&gt;K)&lt;δ P( |X - Y| > K) < \delta 为简单起见,假设: 已知相关系数为“高”,例如: ρX,Y=covar(X,Y)/σXσY≥1−ϵρX,Y=covar(X,Y)/σXσY≥1−ϵ \rho_{X,Y}= {covar(X,Y)} / {\sigma_X \sigma_Y} \geq 1 - \epsilon X,YX,YX,Y 为零均值:μx=μy=0μx=μy=0 \mu_x = \mu_y = 0 −1≤xi,yi≤1−1≤xi,yi≤1-1 \leq x_i, y_i \leq 1(或者 如果这样更容易的话)0≤xi,yi≤10≤xi,yi≤1 0 \leq x_i, y_i \leq 1 (如果它使事情变得容易,那么说具有相同的方差:)X,YX,YX,Y σ2X=σ2YσX2=σY2\sigma_X^2 = \sigma_Y^2 …

3
如何证明
我一直在尝试建立不平等 |Ti|=∣∣Xi−X¯∣∣S≤n−1n−−√|Ti|=|Xi−X¯|S≤n−1n\left| T_i \right|=\frac{\left|X_i -\bar{X} \right|}{S} \leq\frac{n-1}{\sqrt{n}} 其中X¯X¯\bar{X}是样品平均值和SSS样本标准差,即 S=∑ni=1(Xi−X¯)2n−1−−−−−−−−−√S=∑i=1n(Xi−X¯)2n−1S=\sqrt{\frac{\sum_{i=1}^n \left( X_i -\bar{X} \right)^2}{n-1}}。 很容易看到∑ni=1T2i=n−1∑i=1nTi2=n−1\sum_{i=1}^n T_i^2 = n-1 ,因此|Ti|&lt;n−1−−−−−√|Ti|&lt;n−1\left| T_i \right| < \sqrt{n-1}但这与我一直在寻找的目标不是很接近,也不是一个有用的界限。我已经试验了柯西-舒瓦兹(Cauchy-Schwarz)和三角形不等式,但没有成功。我必须在某个地方缺少一个微妙的步骤。谢谢您的帮助。

2
回归结果具有意外的上限
我尝试预测平衡得分,并尝试了几种不同的回归方法。我注意到的一件事是,预测值似乎具有某种上限。也就是说,实际余额为,但我的预测顶部约0.8。下图显示了实际余额与预测余额(通过线性回归预测):[ 0.0 ,1.0 )[0.0,1.0)[0.0, 1.0)0.80.80.8 这是相同数据的两个分布图: 由于我的预测变量非常偏斜(具有幂律分布的用户数据),因此我应用了Box-Cox转换,将结果更改为以下内容: 尽管它改变了预测的分布,但仍然存在上限。所以我的问题是: 预测结果出现上限的可能原因是什么? 如何确定与实际值的分布相对应的预测? 奖励:由于Box-Cox转换后的分布似乎遵循转换后的预测变量的分布,因此这可能直接相关吗?如果是这样,我是否可以应用一种转换以使分布适合实际值? 编辑:我使用了5个预测变量的简单线性回归。
By using our site, you acknowledge that you have read and understand our Cookie Policy and Privacy Policy.
Licensed under cc by-sa 3.0 with attribution required.